36  Números de Carmichael

Seja \(n\) um número natural com \(n\geq 5\) e seja \(a\in\{2,\ldots,n-1\}\). Lembre que \(n\) é dito pseudoprimo na base \(a\) se \(a^{n-1}\equiv 1\pmod n\). Observe que se \(n\) for pseudoprimo na base \(a\), então \(a^n\equiv a\pmod n\).

Definição 36.1 Um número composto \(n\) é dito número de Carmichael se \(a^n\equiv a\pmod n\) para todo \(a\in\{1,\ldots,n-1\}\)

Exercício 36.1 Mostre que um número de Carmichael é ímpar

Números de Carmichael são os mais prováveis de passarem no teste de primalidade de Fermat, pois se \(n\) é de Carmichael, então \(b^{n-1}\equiv 1\pmod n\) vale para qualquer base \(b\) coprimo com \(n\). Ou seja, \(n\) será pseudoprimo para muitas bases.

O menor número de Carmichael é 561. Usando a definição de números de Carmichael, este fato não é fácil de verificar sem usar um computador. No entanto, a verificação pode ser feita sabendo que \[ 561=3\cdot 11\cdot 17. \] Seja \(b\in\{1,\ldots,560\}\) arbitrário. Nós vamos mostrar que \(b^{561}\equiv b\pmod{561}\). Esta congruência é equivalente ao fato que \(561\mid (b^{561}-b)\). Para provar esta divisibilidade, basta verificar que \(3\), \(11\), e \(17\) são divisores de \(b^{561}-b\) para todo \(b\in\{1,\ldots,560\}\).

Primeiro tratamos a afirmação que \(17\mid (b^{561}-b)\); ou seja \[ b^{561}\equiv b\pmod {17}. \] Se \(17\mid b\), então os dois lados da congruência são congruentes com zero, então não há nada para fazer. Assuma que \(17\nmid b\). Neste caso, o Pequeno Teorema de Fermat dá que \(b^{16}\equiv 1\pmod{17}\) e isso implica que \[ b^{560}=(b^{16})^{35}\equiv 1\pmod{17}. \] Obtemos assim que \(b^{561}\equiv b\pmod{17}\) ou seja \(17\mid( b^{561}-b)\).

Notando que \(560=561-1\) é divisível por \(3-1=2\) e por \(11-1=10\), as divisibilidades \(3\mid (b^{561}-b)\) e \(11\mid (p^{561}-b)\) podem ser verificadas similarmente.

Observe que na conta acima os fatos cruciais sobre o número 561 foram os seguintes:

Acontece que estes dois fatos de fato são necessários e suficientes para um número ser de Carmichael.

Teorema 36.1 (Korselt) Seja \(n\geq 2\) um número natural composto. Então \(n\) é um número de Carmichael (ou seja, \(b^n\equiv b\pmod n\) para todo \(b\in\{1,\ldots,n-1\}\)) se e somente se as seguintes propriedades são válidas:

  • \(n\) é livre de quadrado (\(n\) não é divisível por \(p^2\) para nenhum primo \(p\));
  • se \(p\) é um primo que divide \(n\), então \(p-1\) divide \(n-1\).

Comprovação. Vamos começar com a demonstração da volta. Assuma que \(n\in\N\) satisfaz as duas propriedades no teorema. Escreva \(n=p_1\cdots p_k\) onde os \(p_i\) são primos distintos e seja \(a\in\Z\). Precisamos provar que \(a^n\equiv a\pmod n\); ou seja \(n\mid a^n-a\), mas esta última divisibilidade é equivalente às divisibilidades que \(p_i\mid a^n-a\) para todo \(i\).

Se \(p_i\mid a\), então \(p_i\mid a^n-a\). Se \(p_i\nmid a\), então o Pequeno Teorema de Fermat implica que \(a^{p_i-1}\equiv 1\pmod{p_i}\). Pela condição 1., temos que \(p-1\mid n-1\); ou seja \(n=q(p_i-1)+1\) com algum \(q\in\Z\). Logo \[ a^n=a^{q(p_i-1)+1}=(a^{p_i-1})^qa\equiv a\pmod{p_i}. \] Mas isso implica que \(p_i\mid a^n-a\). Como isso é verdadeiro para todo \(i\in\{1,\ldots,k\}\), obtemos que \(n\mid a^n-a\); ou seja \(a^n\equiv a\pmod n\).

Agora consideremos a ida. Assuma que \(n\in\N\) é um número de Carmichael. Promeiro provaremos afirmação 1. Assuma por contradição que \(p^2\mid n\) com algum primo \(p\). Como \(n\) é Carmichael, \(p^n\equiv p\pmod n\), ou seja \(n\mid p^n-p\). Mas isso implica que \[ p^2\mid p^n-p=p(p^{n-1}-1) \] o que é impossível, pois o primeiro fator no lado direito é divisível por \(p\), mas não por \(p^2\), enquanto o segundo fator não é divisível por \(p\).

Agora assuma que \(p\) é um primo tal que \(p\mid n\). Precisamos provar que \(p-1\mid n-1\). O Teorema da Raiz Primitiva implica que existe \(a\in\Z\) tal que \(|a|_p=p-1\) (ou seja, \(a^{p-1}\equiv 1\pmod p\), mas \(a^k\not\equiv 1\pmod p\) para todo \(k\in\{1,\ldots,p-2\}\)). Como \(n\) é Carmichael, temos que \(a^n\equiv a\pmod n\), e assim \(a^n\equiv a\pmod p\) (como \(p\mid n\)). Mas \(a\) é invertível módulo \(p\) e assim \(a^{n-1}\equiv 1\pmod p\) também segue. Isso implica que \[ p-1=|a|_p\mid n-1 \] e é isso que nós queremos provar.